LSAT and Law School Admissions Forum

Get expert LSAT preparation and law school admissions advice from PowerScore Test Preparation.

 Administrator
PowerScore Staff
  • PowerScore Staff
  • Posts: 8919
  • Joined: Feb 02, 2011
|
#23501
Complete Question Explanation

Weaken. The correct answer choice is (B)

This question asks you to address a second hole in the stimulus argument, namely that we do not know the demographic makeup of these two communities. Sure, moving the library to Glenwood may put more people within walking distance, but that is of no consequence if the people in Glenwood do not read. If Redville happens to have a very highly educated populace that is more inclined to take advantage of a library in the area, then the overall argument will be weakened.

Answer Choice (A): Well, of course this is probably true. Since the current library is in Redville, it makes sense that more people who currently walk to the library live in Redville. This statement of basic common sense has no bearing on the stimulus argument.

Answer Choice (B): This is the correct answer choice. This answer choice punches through the above-described hole in the argument and demonstrates that, regardless of how many more people are within walking distance, the demographic makeup of the community will not necessarily lead to more library patrons.

Answer Choice (C): This answer choice clearly strengthens the argument and should be discarded quickly.

Answer Choice (D): This answer choice is irrelevant to the stimulus argument, because the stimulus author is concerned only with putting the library within walking distance of more people. He is not concerned with those individuals who drive or take public transportation.

Answer Choice (E): This answer choice would also strengthen the argument and should be discarded quickly.
 quinnd
  • Posts: 3
  • Joined: Apr 10, 2018
|
#45327
Hey Powerscore Staff,

First, I want to thank you guys for the hard work you put into the forum. I am currently preparing for the July 2018 exam and find myself using this resource daily. It's been so incredibly helpful with respect to picking up all of the nuances of the test, so I truly appreciate it. I'm typically a passive viewer but this question really stung me, so I felt compelled to inquire about it. Here goes!...

I just completed question #35 in Chapter 4 (Weaken) of the LR QTT book and am having a hard time understanding why B is the correct answer. Here is my reasoning, with key points bolded:

The conclusion states that if the library were relocated from R to G, it "would then be within walking distance of a larger number of library users."

This is drawn from the following premises: 1) More people live in G than in R, and 2) People will generally walk to the library only if it is located close to their homes

The credited response (B) states that "If the library were relocated from R to G, the # of people living in G who would use it is less than the # of people living in R who currently use it."

The trouble I am having is in the subtle difference between the conclusion and the credited response (see the bolded text above). The conclusion seems focused on the idea of walking distance, regardless of whether the residents use it or not. However, the correct answer choice discusses the number of current and prospective users.

I don't see how an answer about the anticipated number of users undermines a conclusion regarding walking distance. Even if it were true that fewer residents of G will use the new library than residents of R, it could still be the case that many more of the G residents (albeit non-users) will live within walking distance of the new library, in which case the conclusion would not be weakened. Am I off with my logic here?

I'm usually good about taking my medicine from the testmakers, but I am just not seeing this one. As such, I'm eager to get your explanation. Thanks in advance!

Quinn
 Daniel Stern
PowerScore Staff
  • PowerScore Staff
  • Posts: 81
  • Joined: Feb 07, 2018
|
#45332
Quinn:

You are correct that the conclusion in the stimulus is about walking distance; but it is about the library being within walking distance of library users. This is based on the premise that because town G has more people than town R.

Since answer choice B states that town G has fewer people who will actually use the library, it undermines the conclusion that relocating the library there will put it within walking distance of more library users. Even if, when relocated to town G, the library is now closer to a million people, it doesn't matter if none of those million people actually use the library -- the library then has not been relocated within walking distance of a greater number of library users.

I hope that is helpful, good luck!
Dan
 quinnd
  • Posts: 3
  • Joined: Apr 10, 2018
|
#45346
Thanks, Dan. You pointed out a key but subtle element of the stimulus that I missed: library users. I didn't pay much mind to it and wrongly assumed that the conclusion was referring to all residents, hence my doubt in the efficacy of the correct response. This is why PO #4 is so critical! That's the takeaway for me from this question. Thanks for your help.

Get the most out of your LSAT Prep Plus subscription.

Analyze and track your performance with our Testing and Analytics Package.